Đến nội dung

minhtuyb

minhtuyb

Đăng ký: 19-01-2012
Offline Đăng nhập: 06-05-2015 - 15:29
****-

#382553 $\sqrt{2} + \sqrt{3} + \sqrt{5...

Gửi bởi minhtuyb trong 01-01-2013 - 13:31

Tổng của hai số vô tỉ là một số vô tỉ

Bài cậu đúng rồi nhưng chú ý là mệnh đề trên là một mệnh đề sai nhé. Vd: $1+2\sqrt{2}$và $-2\sqrt{2}$ là hai số vô tỉ nhưng tổng của chúng lại là một số hữu tỉ.

Mệnh đề đúng là: Tổng của hai số vô tỉ không đồng dạng là một số vô tỉ


#382551 $a+b+c\le \dfrac{3}{2}$

Gửi bởi minhtuyb trong 01-01-2013 - 13:26

Đặt $\sin \frac{A}{2} =x;\sin \frac{A}{2} =y;\sin \frac{A}{2} =z;(A;B;C \in (0;\pi))$
Ta có $sin^2 \frac{A}{2}+sin^2 \frac{B}{2}+sin^2 \frac{C}{2}+2sin \frac{B}{2}sin\frac{A}{2}sin \frac{C}{2}=1$
Cái này trên mạng có nhiều rồi
Ta chỉ cần chứng minh $sin \frac{A}{2}+sin \frac{B}{2}+sin \frac{C}{2}\le \frac{3}{2}$

Từ $\sin \frac{A}{2} =x;\sin \frac{A}{2} =y;\sin \frac{A}{2} =z;(A;B;C \in (0;\pi))$ thì suy ra $x^2+y^2+z^2+2xyz=1$. Nhưng liệu từ $x^2+y^2+z^2+2xyz=1$ thì có phải chỉ tồn tại duy nhất cách đặt $\sin \frac{A}{2} =x;\sin \frac{A}{2} =y;\sin \frac{A}{2} =z;(A;B;C \in (0;\pi))$ không?
Cách đặt trên mới chỉ là 1 trường hợp của bài toán :)


#382542 $2\sum a\geq \sum \sqrt[3]{a^{3}+7...

Gửi bởi minhtuyb trong 01-01-2013 - 13:05

Cho $a,b,c,d>0$ thỏa
$a+b+c+d = \frac{1}{a^{2}}+\frac{1}{b^{2}}+\frac{1}{c^{2}}+\frac{1}{d^{2}}$
CMR
$2(a+b+c+d)\geq \sqrt[3]{a^{3}+7}+\sqrt[3]{b^{3}+7}+\sqrt[3]{c^{3}+7}+\sqrt[3]{d^{3}+7}$

Bài này sử dụng $Chebyshev$ là ra :):
---
-Viết lại giả thiết: $\sum \left(a-\dfrac{1}{a^2}\right)=0$. Ta có:
$$bdt\Leftrightarrow \sum (2a-\sqrt[3]{a^3+7})\ge 0\\ \Leftrightarrow \sum \dfrac{8a^3-(a^3+7)}{4a^2+2a\sqrt[3]{a^3+7}+\sqrt[3]{(a^3+7)^2}}\ge 0\\ \Leftrightarrow \sum \dfrac{a-\dfrac{1}{a^2}}{4+2\sqrt[3]{1+\dfrac{7}{a^3}}+\sqrt[3]{(1+\dfrac{7}{a})^2}}\ge 0$$
Giả sử $a\ge b\ge c\ge d$ thì ta có hai dãy đơn điệu cùng chiều:
$$a-\dfrac{1}{a^2}\ge b-\dfrac{1}{b^2}\ge c-\dfrac{1}{c^2}\ge d-\dfrac{1}{d^2}\
\\\dfrac{1}{4+2\sqrt[3]{1+\dfrac{7}{a^3}}+\sqrt[3]{(1+\dfrac{7}{a})^2}}\ge \dfrac{1}{4+2\sqrt[3]{1+\dfrac{7}{b^3}}+\sqrt[3]{(1+\dfrac{7}{b})^2}}\ge ...$$
Áp dụng BĐT $Chebyshev$ cho hai dãy đơn điệu cùng chiều ta có ĐPCM. Dấu bằng xảy ra khi $a=b=c=d=1$


#382536 $a+b+c\le \dfrac{3}{2}$

Gửi bởi minhtuyb trong 01-01-2013 - 12:48

Một bài nho nhỏ mừng năm mới :P:
---
Pro: Khẳng định hoặc phủ định mệnh đề sau: Với số thực không âm $a,b,c$ thỏa $a^2+b^2+c^2+2abc=1$ thì:
$$a+b+c\le \dfrac{3}{2}$$
  • NLT yêu thích


#381618 Đề thi Olympic Bắc Hưng Hải lần thứ nhất - 2012

Gửi bởi minhtuyb trong 29-12-2012 - 17:47

Gọi số lần xuất hiện số 1 là $x_{1}$
số lần xuất hiện số 2 là $x_{2}$
số lần xuất hiện số 3 là $x_{3}$
Ta có :$x_{1}+x_{2}+x_{3}=n$
$\Leftrightarrow x_{1}-1 +x_{2}-1 +x_{3}-1 =n-3(1)$
Theo kết quả bài toán chia kẹo của Euler ta có số nghiệm nguyên của pt (1) là:
$C^{2}_{n-1}$hay có $C^{2}_{n-1}$ số tự nhiên thoả mãn yêu cầu đề bài.

Bài toán chia kẹo không áp dụng được trong trường hợp này bởi vì hai số có các chữ số khác nhau nhưng có thứ tự khác nhau vẫn đc coi là 2 nghiệm khác nhau của bài toán.
---
- Tổng số các số có $n$ chữ số tạo bởi ba chữ số $1,2,3: 3^n$ số
- Số các số có $n$ chữ số mà không bao gồm chữ số 1 (chỉ bao gồm số 2 và 3): $2^n$ số
- Số các số có $n$ chữ số mà không bao gồm chữ số 2 (chỉ bao gồm số 1 và 3): $2^n$ số
- Số các số có $n$ chữ số mà không bao gồm chữ số 3 (chỉ bao gồm số 1 và 2): $2^n$ số
- Số các số có $n$ chữ số mà không bao gồm chữ số 1 và 2: 1 số
- Số các số có $n$ chữ số mà không bao gồm chữ số 2 và 3: 1 số
- Số các số có $n$ chữ số mà không bao gồm chữ số 3 và 1: 1 số
Theo nguyên lí bao gồm và loại trừ thì nghiệm của bài toán là:
$$d=3^n-3(2^n-1)$$


#380252 $$\sqrt{\frac{b+c-a}{a}}+...

Gửi bởi minhtuyb trong 25-12-2012 - 10:43

Giải như sau:
Chuẩn hoá $abc=1$
Qua PP U.C.T ta có thể tìm được BDT phụ sau:
$\frac{a^3+2abc}{a^3+(b+c)^3}\geq \frac{a^2}{a^2+b^2+c^2}$
That vậy điều này tương đương với:
$a^3b^2+a^3c^2+2abc(a^2+b^2+c^2)\geq 3a^2bc(b+c)+b^3a^2+c^3a^2$
$\Leftrightarrow \frac{a^2b}{c}+\frac{a^2c}{b}+2(a^2+b^2+c^2)\geq \frac{b^2a}{c}+\frac{c^2a}{b}+3ab+3ac\Leftrightarrow$
$(a-b)^2(\frac{c}{b}+1)+(a-c)^2(\frac{b}{c}+1)+(b-c)^2(1-\frac{a}{c}-\frac{a}{b})\geq 0$
TH1: $a\geq b\geq c\Rightarrow S_b,S_c>0,a^2Sb+b^2S_a=\frac{a^2b}{c}+a^2+b^2-\frac{ab^2}{c}-ab\geq 0$ ( Dễ thấy)
TH2:$a\leq b\leq c$, ta có:$S_b\geq 0,S_c+S_b\geq 0,S_b+S_a=2-\frac{a}{c}-\frac{a}{b}+\frac{b}{c}\geq 0$ do $\frac{a}{c}+\frac{a}{b}\leq 1+1=2$
Do đó BDT trên đúng
Thiết lập các BDT tương tự rồi cộng vào ta có đpcm
Đẳng thức xảy ra khi và chỉ khi $a=b=c$ hoặc $a=b,c=0$ và các hoán vị.

BĐT trên chỉ đối xứng với $b$ và $c$ thôi nên liệu xét 2 TH trên đã đủ? Mình hiểu ý bạn là "KMTTQ", giả sử $b$ nằm giữa $a,c$ nên xét 2 TH trên. Nhưng trên thực tế là đã mất tính tổng quát :).


#377337 $\sum \dfrac{a}{b+c}\ge \dfrac...

Gửi bởi minhtuyb trong 13-12-2012 - 19:36

Problem: Cho ba số $a,b,c$ không âm. CMR:
$$\dfrac{a}{b+c}+\dfrac{b}{c+a}+\dfrac{c}{a+b}\ge \dfrac{3}{2}+\dfrac{3(a-b)(b-c)(c-a)}{(a+b)(b+c)(c+a)}$$


#374257 $\sum{(-1)^n\frac{1}{2n+2}C_{n...

Gửi bởi minhtuyb trong 01-12-2012 - 17:59

$S_n=1+\sum_{k=1}^n \dfrac{(-1)^k C_n^k}{2k+1}=1+\sum_{k=1}^n\left[\dfrac{n}{n-k}.\dfrac{(-1)^k C_{n-1}^k}{(2k+1)}\right]$

Vì $k$ chạy từ $1$ đến $n$ nên biểu thức $\dfrac{n}{n-k}$ không có nghĩa khi $n=k$.
Cách khắc phục: Tính riêng tổng khi $k=n$ (giống $k=0$) và chỉ cho $k=\overline{2,k-1}$
---
Em nghĩ thế :)


#372956 BĐT AM-GM

Gửi bởi minhtuyb trong 26-11-2012 - 22:57

bài nữa nhé
Bài 7: Cho $a,b,c>0$. CMR
$\frac{a^4}{b^2(b+c)}+\frac{b^4}{c^2(c+a)}+\frac{c^4}{a^(a+b)}\geq \frac{a+b+c}{2}$ :icon6:


Đề nghị các bạn giải quyết hết bài tồn động trước khi quăng bài mới :ph34r:
---
$$\dfrac{a^4}{a^2(b+c)}+\dfrac{a}{2}+\dfrac{a}{2}+\dfrac{b+c}{4}\ge 2a$$
Tương tự rồi cộng lại

Do phần này khá rộng nên mình xin trình bày bài tập riêng (hơi nhiều 1 tí) :icon6:
1,Chứng minh với mọi a,b,c $0\leq a,b,c\leq \frac{1}{2}$ thỏa mãn $a+b+c=1$ thì
$\frac{1}{a(2b+2c-1)}+\frac{1}{b(2c+2a-1)}+\frac{1}{c(2a+2b-1)}\geq 27$



Bài này mà $AM-GM$ được thì cũng hơi xoắn :o
---
$$\frac{1}{a(2b+2c-1)}=\dfrac{1}{a(1-2a)}$$
Sau đó ta đi c/m BĐT phụ:
$$\dfrac{1}{a(1-2a)}\ge 27a\\ \Leftrightarrow (3a-1)^2(6a+1)\ge 0$$

2,Chứng minh với mọi a,b,c $\geq \frac{1}{2}$,ta có
$\frac{a^2}{\sqrt{5-2(b+c)}}+\frac{b^2}{\sqrt{5-2(c+a)}}+\frac{c^2}{\sqrt{5-2(a+b)}}\geq 3$


$$\dfrac{a^2}{\sqrt{5-2(b+c)}}\ge \dfrac{(\dfrac{1}{2})^2}{\sqrt{5-2(\dfrac{1}{2}+\dfrac{1}{2})}}=\dfrac{\sqrt{3}}{12}$$
Tương tự rồi cộng lại ta thấy $VT\ge \dfrac{\sqrt{3}}{4}$

3,Chứng minh với mọi a,b,c có $0< a,b,c\leq \frac{1}{3},a^3+b^3+c^3=\frac{3}{64}$ thì
$\frac{1}{1-3a}+\frac{1}{1-3b}+\frac{1}{1-3c}\geq 12$


Tư tưởng tương tự bài $2$, ta xây dựng BĐT phụ:
$$\dfrac{1}{1-3a}\ge 256a^3\\ \Leftrightarrow (4a-1)^2(48a^2+8a+1)\ge 0\ (True)$$


#372715 Cho $a, b, c >0$ và $ab+bc+ca=abc$. Chứng minh rằng:...

Gửi bởi minhtuyb trong 26-11-2012 - 08:59

Cho $a, b, c >0$ và $ab+bc+ca=abc$.
Chứng minh rằng: $\sum \frac{\sqrt{b^2+2a^2}}{ab}\geq \sqrt{3}$

$$LHS.\sqrt{3}=\sum \dfrac{\sqrt{(b^2+2a^2)(1+2)}}{ab}\ge^{Cauchy} \sum \dfrac{\sqrt{(b+2a)^2}}{ab}=\sum \dfrac{b+2a}{ab}=\sum \left(\dfrac{1}{a}+\dfrac{2}{b}\right)$$
Mà theo giả thiết thì $\sum \dfrac{1}{a}=1\Rightarrow LHS.\sqrt{3}\ge 3\Rightarrow LHS\ge \sqrt{3}=RHS$
Dấu bằng xảy ra khi $a=b=c=3\ \square$
---
P/s: Bài này cũng không cần giả thiết $a,b,c>0$


#372533 Một số bất đẳng thức quan trọng

Gửi bởi minhtuyb trong 25-11-2012 - 17:47

Nhầm box nghiêm trọng Hình đã gửi


#372525 BĐT AM-GM

Gửi bởi minhtuyb trong 25-11-2012 - 17:31

a,Hãy suy xét một bài toán cơ bản
Ta xét BĐT sau:Với mọi a,b,c dương có tổng bằng 3
$\frac{1}{a^2(1+a)}+\frac{1}{b^2(1+b)}+\frac{1}{c^2(1+c)}\geq \frac{3}{4abc}$(Olimpic 30-4)
Giải:Ta hoàn toàn có thể giải quyết bài toán trên bằng đánh giá thuận C-S,tuy nhiên,ta sẽ xem xét 1 lời giải sau
BĐT
$\Leftrightarrow \frac{abc}{a^2(1+a)}+\frac{abc}{b^2(1+b)}+\frac{abc}{c^2(1+c)}\geq \frac{3}{4}$
$\Leftrightarrow \frac{bc(b+c)}{a\left [ (b+c)^2+2a(b+c) \right ]} +\frac{ca(c+a)}{b\left [ (c+a)^2+2b(c+a) \right ]}+\frac{ab(a+b)}{c\left [ (a+b)^2+2c(a+b) \right ]}\geq \frac{3}{4}$
$\Leftrightarrow \frac{bc(b+c)}{2a\left [ (b^2+c^2+a(b+c) \right ]} +\frac{ca(c+a)}{2b\left [ a^2+c^2+b(c+a) \right ]}+\frac{ab(a+b)}{2c\left [ a^2+b^2+c(a+b) \right ]}\geq \frac{3}{4}$
$\Leftrightarrow \frac{(b+c)}{2a(\frac{a+b}{c}+\frac{a+c}{b})} +\frac{(c+a)}{2b(\frac{b+c}{a}+\frac{a+b}{c})}+\frac{(a+b)}{2c(\frac{b+c}{a}+\frac{c+a}{b})}\geq \frac{3}{4}$

Bằng việc đặt $\frac{a+b}{c}=x,\frac{b+c}{a}=y,\frac{c+a}{b}=z$,ta dễ dàng đưa BĐT trên về nesbitt 3 biến

Yêu cầu cậu đặt lại phép suy luận toán học ở bài này! Có chỗ là dấu $\Leftarrow$ chứ không phải $\Leftrightarrow$.

c,Làm mạnh BĐT

Tư tưởng giải 2 bài này giống như bài trên nên các bạn luyện tập thêm
"Cũng chẳng có gì khó cả,cái này bình thường thôi"-tôi tin là bạn đang nghĩ vậy,tuy nhiên,bạn hãy xem thêm 2 VD sau để chắc chắn cái bạn nói là đúng nhé
Từ BĐT AM-GM 3 số,ta có thể làm mạnh lên thành
$\frac{a+b+c}{3}\geq \sqrt[3]{abc}+\frac{(a-b)^2+(b-c)^2+(c-a)^2}{12(a+b+c)}$
Cũng cần nói thêm là BĐT này cần tới S.O.S để giải (quá ảo) :lol:

Chưa thử c/m BĐT này bằng $S.O.S$. Nhưng mình cũng xin giới thiệu một BĐT khác cũng khá giống cái này, nhưng được làm mạnh trực tiếp từ $AM-GM$:
$\dfrac{a+b+c}{3}\geq \sqrt[3]{abc}+\dfrac{(\sqrt{a}-\sqrt{b})^2+(\sqrt{b}-\sqrt{c})^2+(\sqrt{c}-\sqrt{a})^2}{3}$

Qủa này là hệ số bất định phải không minhtuyb hầy? :wacko:

Cũng không hẳn. Mình dùng pp tiếp tuyến thôi cho nhanh (bấm máy ra luôn đạo hàm tại $x=1$ là $0$ ) :D


#372298 BĐT AM-GM

Gửi bởi minhtuyb trong 25-11-2012 - 00:17

Mình xin đóng góp cho topic bài này
Bài 5: Cho $a,b,c>0$ và $a+b+c=3$. CMR
$\sum \frac{x}{x^4+y+z}\geq 1$

Mình lại c/m được $\sum \frac{x}{x^4+y+z}\leq 1\ (*)$ Hình đã gửi
---
Vời giả thiết trên thì: $\dfrac{x}{x^4+y+z}=\dfrac{x}{x^4+3-x}$
Ta sẽ c/m: $\dfrac{x}{x^4+3-x}\le \dfrac{1}{3}\Leftrightarrow x^4-4x+3\ge 0\Leftrightarrow (x-1)^2(x^2+2x+3)\ge 0$ (Luôn đúng)
C/m tt rồi cộng lại ta suy ra $(*)$ đúng.
Dấu bằng xảy ra khi $x=y=z=1\ \square$
---
Thôi đi ngủ đây, lần sau bạn nhớ post bài nào liên quan tới topic nhé Hình đã gửi


#372296 BĐT AM-GM

Gửi bởi minhtuyb trong 25-11-2012 - 00:09

c,Đặt ẩn phụ
Vì kĩ thuật đặt ăn phụ mình đã trình bày nên xin chỉ nêu 1 VD nho nhỏ sau
Chứng mnih với mọi a,b,c dương,ta có
$\frac{1}{a\sqrt{a+b}}+\frac{1}{b\sqrt{b+c}}+\frac{1}{c\sqrt{c+a}}\geq \frac{3}{\sqrt{2abc}}$
Giải BĐT cần chứng minh tương đương $\sqrt{\frac{2bc}{a(a+b)}}+\sqrt{\frac{2ca}{b(b+c)}}+\sqrt{\frac{2ab}{c(c+a)}}\geq 3$
Ta sẽ đặt $\sqrt{\frac{2bc}{a(a+b)}}=x;\sqrt{\frac{2ca}{b(b+c)}}=y;\sqrt{\frac{2ab}{c(c+a)}}=z$
Chú ý thêm rằng xy=$\frac{2c}{\sqrt{(a+b)(b+c)}}$,ta sẽ chứng minh $xy+yz+zx\geq 3$
Tiếp tục đặt $\sqrt{a+b}=p,\sqrt{b+c}=q,\sqrt{c+a}=r$,khi đó,dề thấy $xy=\frac{p^2+q^2-r^2}{qr}$
BĐT cần chứng minh sẽ là $(p^3+q^3+r^3)(p^2q+q^2r+r^2p)\geq q^2p+r^2q+p^2r+3pqr$(Đây là 1 BĐT khá hay,xin nhường cho các bạn)

Hix $p,q,r$ trâu bò quá ="=. Mình xin đóng góp lời giải theo mình nghĩ là hay hơn :lol: :
---
BĐT cần chứng minh tương đương:
$$\sum \sqrt{\frac{2bc}{a(a+b)}}\geq 3\ (*)$$
Ta có:
$$LHS(*)=\sum \dfrac{2bc}{\sqrt{2abc(a+b)}}=\sum \dfrac{2bc}{\sqrt{2ab(ac+bc)}}\ge \sum \dfrac{4bc}{2ab+ac+bc}$$
Đặt $x=ab;y=bc;z=ca\Rightarrow x,y,z>0$. Ta cần c/m:
$$\sum \dfrac{4x}{x+2y+z}\ge 3\Leftrightarrow \sum \dfrac{x}{x+2y+z}\ge \dfrac{3}{4}\ (**)$$

(Đến đây nếu dùng $Schwarz$ thì ra luôn. Nhưng vì là topic $AM-GM$ nên ta xài luôn $AM-GM$ :icon6: )

Chuẩn hoá: $x+y+z=3$. Viết lại $(**)$:
$$\sum \dfrac{x}{y+3}\ge \dfrac{3}{4}$$
Đúng từ các BĐT sau:
$$\dfrac{x}{y+3}+\dfrac{x(y+3)}{16}\ge \dfrac{x}{2}\\ \dfrac{y}{z+3}+\dfrac{y(z+3)}{16}\ge \dfrac{y}{2}\\\dfrac{z}{x+3}+\dfrac{z(x+3)}{16}\ge \dfrac{z}{2}\\ \dfrac{(x+y+z)^2}{3.16}\ge \dfrac{xy+yz+zx}{16}$$
Vậy $(**)$ đúng $\Rightarrow (*)$ đúng.
Dấu bằng xảy ra khi $a=b=c$
Phép c/m hoàn tất $\square$.


#372270 BĐT AM-GM

Gửi bởi minhtuyb trong 24-11-2012 - 23:06

Nhìn phát thấy ngay là $Holder$ Hình đã gửi
---
-Nếu $\exists a_i+b_i=0\Rightarrow a_i=b_i=0\Rightarrow VT(*)=VP(*)$
-Nếu các biến đều dương: áp dụng BĐT $AM-GM$ cho $n$ số, ta có:
$$\dfrac{a_1}{a_1+b_1}+\dfrac{a_2}{a_2+b_2}+...+\dfrac{a_n}{a_n+b_n}\ge \dfrac{n\sqrt[n]{a_1a_2...a_n}}{(a_{1}+b_{1})(a_{2}+b_{2})...(a_{n}+b_{n})}\\
\dfrac{b_1}{a_1+b_1}+\dfrac{b_2}{a_2+b_2}+...+\dfrac{b_n}{a_n+b_n}\ge \dfrac{n\sqrt[n]{b_1b_2...b_n}}{(a_{1}+b_{1})(a_{2}+b_{2})...(a_{n}+b_{n})}$$
Cộng vế với vế của 2 BĐT cùng chiều trên, sau vài bước biến đổi ta có ĐPCM. Dấu bằng xảy ra chẳng hạn khi $a_1=a_2=...=a_n;b_1=b_2=...=b_n\ \square$
---
Hai bài trên là hệ quả trực tiếp của BĐT này

mọi người ta tập trung lại 1 tí nhé,mình post bài có kèm theo bài tập là muốn mọi người vận dụng thẳng các kiến thức trên vào bài tập cho vững luôn,dĩ nhiên mình không nề nà gì việc đăng thêm bài mới,nhưng mình nghĩ,việc đó nên để khi ta đã giải quyết trọn vẹn các bài tập
thân
tình hình là ta còn nhiều bài tồn đọng quá
ĐỀ NGHỊ
bài 4 phần 1(kiến thức đã nêu,mong mọi người triệt để áp dụng :wacko:
với mọi a.b không âm,chứng minh
$\sqrt[3]{\frac{a}{b}}+\sqrt[3]{\frac{b}{a}}\leq \sqrt[3]{2(a+b)(\frac{1}{a}+\frac{1}{b})}$

Ô đề nghị này ko ai làm á :ohmy:
---
Áp dụng cái mình c/m đằng trước với $n=3$, ta có:
$$RHS=\sqrt[3]{(1+1)(a+b)(\dfrac{1}{b}+\dfrac{1}{a})}\ge \sqrt[3]{1.a.\dfrac{1}{b}}+\sqrt[3]{1.b.\dfrac{1}{a} }\\ \Rightarrow Q.E.D$$
---
Mn ủng hộ topic đi chứ nhỉ ~O)